Stay ahead of learning milestones! Enroll in a class over the summer!

G
Topic
First Poster
Last Poster
k a May Highlights and 2025 AoPS Online Class Information
jlacosta   0
May 1, 2025
May is an exciting month! National MATHCOUNTS is the second week of May in Washington D.C. and our Founder, Richard Rusczyk will be presenting a seminar, Preparing Strong Math Students for College and Careers, on May 11th.

Are you interested in working towards MATHCOUNTS and don’t know where to start? We have you covered! If you have taken Prealgebra, then you are ready for MATHCOUNTS/AMC 8 Basics. Already aiming for State or National MATHCOUNTS and harder AMC 8 problems? Then our MATHCOUNTS/AMC 8 Advanced course is for you.

Summer camps are starting next month at the Virtual Campus in math and language arts that are 2 - to 4 - weeks in duration. Spaces are still available - don’t miss your chance to have an enriching summer experience. There are middle and high school competition math camps as well as Math Beasts camps that review key topics coupled with fun explorations covering areas such as graph theory (Math Beasts Camp 6), cryptography (Math Beasts Camp 7-8), and topology (Math Beasts Camp 8-9)!

Be sure to mark your calendars for the following upcoming events:
[list][*]May 9th, 4:30pm PT/7:30pm ET, Casework 2: Overwhelming Evidence — A Text Adventure, a game where participants will work together to navigate the map, solve puzzles, and win! All are welcome.
[*]May 19th, 4:30pm PT/7:30pm ET, What's Next After Beast Academy?, designed for students finishing Beast Academy and ready for Prealgebra 1.
[*]May 20th, 4:00pm PT/7:00pm ET, Mathcamp 2025 Qualifying Quiz Part 1 Math Jam, Problems 1 to 4, join the Canada/USA Mathcamp staff for this exciting Math Jam, where they discuss solutions to Problems 1 to 4 of the 2025 Mathcamp Qualifying Quiz!
[*]May 21st, 4:00pm PT/7:00pm ET, Mathcamp 2025 Qualifying Quiz Part 2 Math Jam, Problems 5 and 6, Canada/USA Mathcamp staff will discuss solutions to Problems 5 and 6 of the 2025 Mathcamp Qualifying Quiz![/list]
Our full course list for upcoming classes is below:
All classes run 7:30pm-8:45pm ET/4:30pm - 5:45pm PT unless otherwise noted.

Introductory: Grades 5-10

Prealgebra 1 Self-Paced

Prealgebra 1
Tuesday, May 13 - Aug 26
Thursday, May 29 - Sep 11
Sunday, Jun 15 - Oct 12
Monday, Jun 30 - Oct 20
Wednesday, Jul 16 - Oct 29

Prealgebra 2 Self-Paced

Prealgebra 2
Wednesday, May 7 - Aug 20
Monday, Jun 2 - Sep 22
Sunday, Jun 29 - Oct 26
Friday, Jul 25 - Nov 21

Introduction to Algebra A Self-Paced

Introduction to Algebra A
Sunday, May 11 - Sep 14 (1:00 - 2:30 pm ET/10:00 - 11:30 am PT)
Wednesday, May 14 - Aug 27
Friday, May 30 - Sep 26
Monday, Jun 2 - Sep 22
Sunday, Jun 15 - Oct 12
Thursday, Jun 26 - Oct 9
Tuesday, Jul 15 - Oct 28

Introduction to Counting & Probability Self-Paced

Introduction to Counting & Probability
Thursday, May 15 - Jul 31
Sunday, Jun 1 - Aug 24
Thursday, Jun 12 - Aug 28
Wednesday, Jul 9 - Sep 24
Sunday, Jul 27 - Oct 19

Introduction to Number Theory
Friday, May 9 - Aug 1
Wednesday, May 21 - Aug 6
Monday, Jun 9 - Aug 25
Sunday, Jun 15 - Sep 14
Tuesday, Jul 15 - Sep 30

Introduction to Algebra B Self-Paced

Introduction to Algebra B
Tuesday, May 6 - Aug 19
Wednesday, Jun 4 - Sep 17
Sunday, Jun 22 - Oct 19
Friday, Jul 18 - Nov 14

Introduction to Geometry
Sunday, May 11 - Nov 9
Tuesday, May 20 - Oct 28
Monday, Jun 16 - Dec 8
Friday, Jun 20 - Jan 9
Sunday, Jun 29 - Jan 11
Monday, Jul 14 - Jan 19

Paradoxes and Infinity
Mon, Tue, Wed, & Thurs, Jul 14 - Jul 16 (meets every day of the week!)

Intermediate: Grades 8-12

Intermediate Algebra
Sunday, Jun 1 - Nov 23
Tuesday, Jun 10 - Nov 18
Wednesday, Jun 25 - Dec 10
Sunday, Jul 13 - Jan 18
Thursday, Jul 24 - Jan 22

Intermediate Counting & Probability
Wednesday, May 21 - Sep 17
Sunday, Jun 22 - Nov 2

Intermediate Number Theory
Sunday, Jun 1 - Aug 24
Wednesday, Jun 18 - Sep 3

Precalculus
Friday, May 16 - Oct 24
Sunday, Jun 1 - Nov 9
Monday, Jun 30 - Dec 8

Advanced: Grades 9-12

Olympiad Geometry
Tuesday, Jun 10 - Aug 26

Calculus
Tuesday, May 27 - Nov 11
Wednesday, Jun 25 - Dec 17

Group Theory
Thursday, Jun 12 - Sep 11

Contest Preparation: Grades 6-12

MATHCOUNTS/AMC 8 Basics
Friday, May 23 - Aug 15
Monday, Jun 2 - Aug 18
Thursday, Jun 12 - Aug 28
Sunday, Jun 22 - Sep 21
Tues & Thurs, Jul 8 - Aug 14 (meets twice a week!)

MATHCOUNTS/AMC 8 Advanced
Sunday, May 11 - Aug 10
Tuesday, May 27 - Aug 12
Wednesday, Jun 11 - Aug 27
Sunday, Jun 22 - Sep 21
Tues & Thurs, Jul 8 - Aug 14 (meets twice a week!)

AMC 10 Problem Series
Friday, May 9 - Aug 1
Sunday, Jun 1 - Aug 24
Thursday, Jun 12 - Aug 28
Tuesday, Jun 17 - Sep 2
Sunday, Jun 22 - Sep 21 (1:00 - 2:30 pm ET/10:00 - 11:30 am PT)
Monday, Jun 23 - Sep 15
Tues & Thurs, Jul 8 - Aug 14 (meets twice a week!)

AMC 10 Final Fives
Sunday, May 11 - Jun 8
Tuesday, May 27 - Jun 17
Monday, Jun 30 - Jul 21

AMC 12 Problem Series
Tuesday, May 27 - Aug 12
Thursday, Jun 12 - Aug 28
Sunday, Jun 22 - Sep 21
Wednesday, Aug 6 - Oct 22

AMC 12 Final Fives
Sunday, May 18 - Jun 15

AIME Problem Series A
Thursday, May 22 - Jul 31

AIME Problem Series B
Sunday, Jun 22 - Sep 21

F=ma Problem Series
Wednesday, Jun 11 - Aug 27

WOOT Programs
Visit the pages linked for full schedule details for each of these programs!


MathWOOT Level 1
MathWOOT Level 2
ChemWOOT
CodeWOOT
PhysicsWOOT

Programming

Introduction to Programming with Python
Thursday, May 22 - Aug 7
Sunday, Jun 15 - Sep 14 (1:00 - 2:30 pm ET/10:00 - 11:30 am PT)
Tuesday, Jun 17 - Sep 2
Monday, Jun 30 - Sep 22

Intermediate Programming with Python
Sunday, Jun 1 - Aug 24
Monday, Jun 30 - Sep 22

USACO Bronze Problem Series
Tuesday, May 13 - Jul 29
Sunday, Jun 22 - Sep 1

Physics

Introduction to Physics
Wednesday, May 21 - Aug 6
Sunday, Jun 15 - Sep 14
Monday, Jun 23 - Sep 15

Physics 1: Mechanics
Thursday, May 22 - Oct 30
Monday, Jun 23 - Dec 15

Relativity
Mon, Tue, Wed & Thurs, Jun 23 - Jun 26 (meets every day of the week!)
0 replies
jlacosta
May 1, 2025
0 replies
Thailand MO 2025 P2
Kaimiaku   1
N 14 minutes ago by totalmathguy
A school sent students to compete in an academic olympiad in $11$ differents subjects, each consist of $5$ students. Given that for any $2$ different subjects, there exists a student compete in both subjects. Prove that there exists a student who compete in at least $4$ different subjects.
1 reply
Kaimiaku
an hour ago
totalmathguy
14 minutes ago
Sum and product of digits
Sadigly   5
N 18 minutes ago by Bergo1305
Source: Azerbaijan NMO 2018
For a positive integer $n$, define $f(n)=n+P(n)$ and $g(n)=n\cdot S(n)$, where $P(n)$ and $S(n)$ denote the product and sum of the digits of $n$, respectively. Find all solutions to $f(n)=g(n)$
5 replies
Sadigly
Sunday at 9:19 PM
Bergo1305
18 minutes ago
Anything real in this system must be integer
Assassino9931   4
N 18 minutes ago by Leman_Nabiyeva
Source: Al-Khwarizmi International Junior Olympiad 2025 P1
Determine the largest integer $c$ for which the following statement holds: there exists at least one triple $(x,y,z)$ of integers such that
\begin{align*} x^2 + 4(y + z) = y^2 + 4(z + x) = z^2 + 4(x + y) = c \end{align*}and all triples $(x,y,z)$ of real numbers, satisfying the equations, are such that $x,y,z$ are integers.

Marek Maruin, Slovakia
4 replies
+1 w
Assassino9931
May 9, 2025
Leman_Nabiyeva
18 minutes ago
Oh my god
EeEeRUT   1
N 25 minutes ago by ItzsleepyXD
Source: TMO 2025 P5
In a class, there are $n \geqslant 3$ students and a teacher with $M$ marbles. The teacher then play a Marble distribution according to the following rules. At the start, the teacher distributed all her marbles to students, so that each student receives at least $1$ marbles from the teacher. Then, the teacher chooses a student , who has never been chosen before, such that the number of marbles that he owns in a multiple of $2(n-1)$. That chosen student then equally distribute half of his marbles to $n-1$ other students. The same goes on until the teacher is not able to choose anymore student.

Find all integer $M$, such that for some initial numbers of marbles that the students receive, the teacher can choose all the student(according to the rule above), so that each student receiving equal amount of marbles at the end.
1 reply
EeEeRUT
2 hours ago
ItzsleepyXD
25 minutes ago
No more topics!
Concurrence in Cyclic Quadrilateral
GrantStar   39
N May 3, 2025 by ItsBesi
Source: IMO Shortlist 2023 G3
Let $ABCD$ be a cyclic quadrilateral with $\angle BAD < \angle ADC$. Let $M$ be the midpoint of the arc $CD$ not containing $A$. Suppose there is a point $P$ inside $ABCD$ such that $\angle ADB = \angle CPD$ and $\angle ADP = \angle PCB$.

Prove that lines $AD, PM$, and $BC$ are concurrent.
39 replies
GrantStar
Jul 17, 2024
ItsBesi
May 3, 2025
Concurrence in Cyclic Quadrilateral
G H J
G H BBookmark kLocked kLocked NReply
Source: IMO Shortlist 2023 G3
The post below has been deleted. Click to close.
This post has been deleted. Click here to see post.
GrantStar
821 posts
#1 • 9 Y
Y by OronSH, peace09, MarkBcc168, Rounak_iitr, centslordm, NonoPL, ihatemath123, Funcshun840, ItsBesi
Let $ABCD$ be a cyclic quadrilateral with $\angle BAD < \angle ADC$. Let $M$ be the midpoint of the arc $CD$ not containing $A$. Suppose there is a point $P$ inside $ABCD$ such that $\angle ADB = \angle CPD$ and $\angle ADP = \angle PCB$.

Prove that lines $AD, PM$, and $BC$ are concurrent.
Z K Y
The post below has been deleted. Click to close.
This post has been deleted. Click here to see post.
youlost_thegame_1434
32 posts
#2
Y by
grant sniped
Z K Y
The post below has been deleted. Click to close.
This post has been deleted. Click here to see post.
GrantStar
821 posts
#3 • 8 Y
Y by OronSH, peace09, centslordm, NonoPL, Zhaom, ehuseyinyigit, Funcshun840, ihatemath123
Let $AC$ intersect $DP$ and $BM$ and $X$ and $Z$, and let $BD$ intersect $AM$ and $CP$ at $Y$ and $W$. Also, let $AD$ and $BC$ meet at $E$. By the second angle condition, $CWXD$ is cyclic. The arc midpoint also gives $BZYA$ is cyclic.

https://cdn.artofproblemsolving.com/attachments/c/e/f4adcd5b37f55d49cad3134896766a026e84e5.png

Claim: $BWZC$ and $DYXA$ are cyclic.
Proof. We prove one and symmetry finishes. Note that $\angle AMB=\angle CPD$ and
\[\angle MAD+\angle ADP=\angle MBC+\angle BCP\]so $PD\parallel BM$ and $AM \parallel CP$. Thus
\[\measuredangle XAY=\measuredangle CAM=\measuredangle ACP=\measuredangle XCW=\measuredangle XDW=\measuredangle XDY
\]as desired. $\blacksquare$

Now let the circles above be $\omega_1$ and $\omega_2$. Radical axis on $\omega_1, \omega_2$, and taking turns between $(CWXD)$, $(BZYA)$, and $(ABCD)$ finishes since all of $P,M,E$ lie on the radical axis of $\omega_2$ and $\omega_2$.
Z K Y
The post below has been deleted. Click to close.
This post has been deleted. Click here to see post.
OronSH
1747 posts
#4 • 2 Y
Y by peace09, centslordm
[asy]
import graph; size(10cm); 
real labelscalefactor = 0.5;
pen dps = linewidth(0.7) + fontsize(10); defaultpen(dps);
pen dotstyle = black;
real xmin = -23.88782587247942, xmax = 28.11307823832208, ymin = -9.242139859667088, ymax = 6.863695719095153;

draw(circle((0.5809291498702329,-0.01981345647294291), 5.292133555959725), linewidth(1)); 
draw((0.5542578591817454,5.272252890084698)--(12.770695651820194,-1.2314621937659902), linewidth(0.6)); 
draw((-1.172584231738878,-5.012995647684546)--(12.770695651820194,-1.2314621937659902), linewidth(0.6)); 
draw((-2.6235032789327573,4.1918715250220755)--(5.864154960092957,-0.3267378971930568), linewidth(0.6)); 
draw((-3.8232675432202528,-2.954047438739143)--(5.864154960092957,-0.3267378971930568), linewidth(0.6)); 
draw((-2.6235032789327573,4.1918715250220755)--(4.616982119434293,-3.4428171083827763), linewidth(0.6)); 
draw((-3.8232675432202528,-2.954047438739143)--(4.986206114864625,2.9127984230305017), linewidth(0.6)); 
draw((2.672898183950598,1.372204559170503)--(2.52153455609322,-1.2332843473134645), linewidth(0.6)); 
draw((4.986206114864625,2.9127984230305017)--(4.616982119434293,-3.4428171083827763), linewidth(0.6)); 
draw((1.0656729475516833,0.30184064265604144)--(12.770695651820194,-1.2314621937659902), linewidth(0.6) + linetype("4 4")); 
dot((-1.172584231738878,-5.012995647684546),dotstyle); 
label("$A$", (-1.6429946695254456,-5.883748135844466), NE * labelscalefactor); 
dot((0.5542578591817454,5.272252890084698),dotstyle); 
label("$B$", (0.4153744515271139,5.672008333222611), NE * labelscalefactor); 
dot((4.986206114864625,2.9127984230305017),dotstyle); 
label("$C$", (5.146012256051417,3.2886335614775266), NE * labelscalefactor); 
dot((4.616982119434293,-3.4428171083827763),dotstyle); 
label("$D$", (4.568224432598067,-4.294831621347743), NE * labelscalefactor); 
dot((12.770695651820194,-1.2314621937659902),linewidth(4pt) + dotstyle); 
label("$Z$", (13.01837135060331,-1.4420042430468083), NE * labelscalefactor); 
dot((5.864154960092957,-0.3267378971930568),linewidth(4pt) + dotstyle); 
label("$M$", (6.084917469163111,-0.06975816234509281), NE * labelscalefactor); 
dot((-2.6235032789327573,4.1918715250220755),linewidth(4pt) + dotstyle); 
label("$X$", (-3.3041346619538268,4.4080974694184), NE * labelscalefactor); 
dot((-3.8232675432202528,-2.954047438739143),linewidth(4pt) + dotstyle); 
label("$Y$", (-4.604157264723864,-3.4642616251335467), NE * labelscalefactor); 
dot((1.0656729475516833,0.30184064265604144),linewidth(4pt) + dotstyle); 
label("$P$", (0.7764918411854577,-0.5392107689009429), NE * labelscalefactor); 
dot((2.672898183950598,1.372204559170503),linewidth(4pt) + dotstyle); 
label("$E$", (2.5098553115455076,1.7719405249124727), NE * labelscalefactor); 
dot((2.52153455609322,-1.2332843473134645),linewidth(4pt) + dotstyle); 
label("$F$", (2.257073138784667,-2.019792066500162), NE * labelscalefactor); 
clip((xmin,ymin)--(xmin,ymax)--(xmax,ymax)--(xmax,ymin)--cycle); 
[/asy]

Let $BC\cap AD=Z.$ Let $DP,CP$ meet $\omega$ again at $X,Y$ respectively.

We claim $BC\parallel MX$ and $AD\parallel MY.$ This follows from angle chasing, since the first condition gives that arcs $BX$ and $AY$ sum to arc $CD,$ and the second gives that arcs $BX$ and $AY$ are equal. Thus arc $BX$ equals arc $CM,$ and arc $AY$ equals arc $DM.$

Now let $E=MX\cap CY$ and $F=MY\cap DX.$ Pascal's on $CYMMXD$ gives $EF\parallel CD,$ since the tangent to $\omega$ at $M$ is parallel to $CD.$

Finally, we have $EM\parallel CZ,FM\parallel DZ,EF\parallel CD$ so $\triangle EFM$ and $\triangle CDZ$ are homothetic. The center of homothety must be $CE\cap FD=P,$ so $P,M,Z$ collinear as desired.
Z K Y
The post below has been deleted. Click to close.
This post has been deleted. Click here to see post.
MarkBcc168
1595 posts
#5 • 5 Y
Y by OronSH, peace09, centslordm, ehuseyinyigit, Akram_Lahlou
I like radical axis.

[asy]
size(7.5cm);
defaultpen(fontsize(10pt));
import olympiad;
import geometry;
pair A = dir(-20);
pair B = dir(-130);
pair C = dir(110);
pair D = dir(70);
pair M = dir(90);
pair X = 2*foot(-M,C,B) - C;
pair Y = 2*foot(-M,A,D) - D;
pair P = extension(C,Y,D,X);
pair T = 2*foot((0,0),M,P) - M;
draw(circle(A,B,C), linewidth(0.7));
draw(A--B--C--D--cycle, linewidth(1));
draw(C--Y--A, black);
draw(D--X--B, black);
draw(arc(circumcenter(C,X,P),circumradius(C,X,P),70,-80,CW), gray+0.7);
draw(arc(circumcenter(D,Y,P),circumradius(D,Y,P),130,-90,CCW), gray+0.7);
dot("$A$", A, dir(5));
dot("$B$", B, dir(-163));
dot("$C$", C, dir(75));
dot("$D$", D, dir(111));
dot("$M$", M, dir(89));
dot("$X$", X, dir(-66));
dot("$Y$", Y, dir(-100));
dot("$P$", P, dir(7));
dot("$T$", T, 1.5*dir(-19));
[/asy]
Let $X = DP\cap BC$ and $Y=CP\cap AD$, so the angle condition implies $\odot(CDXY)$ is cyclic. Consider circles $\odot(CPX)$ and $\odot(DPY)$. Clearly, $AD\cap BC$ lies on the radical axis, so it suffices to show that $M$ does.

To that end, let $T$ be the intersection of those circles. Observe that
\begin{align*}\angle CTD &= \angle CTP + \angle DTP
= \angle CXP + \angle DYP \\
&= \angle(BC, AD) - \angle CPD
= \angle(BC, AD) - \angle BDA \\
&= 180^\circ - \angle CMD,
\end{align*}so $T\in\odot(ABCD)$. Moreover, $\angle CTP = \angle CXP = \angle DYP = \angle DTP$, so $M\in PT$. This implies the conclusion.
Z K Y
The post below has been deleted. Click to close.
This post has been deleted. Click here to see post.
AlephG_64
52 posts
#6 • 4 Y
Y by khina, SomeonesPenguin, NonoPL, ehuseyinyigit
Claim 1: $PD \parallel BM$ and $AM \parallel PC$
Proof:
We know $\angle DPC = \angle AMB$, also $\angle MAD = \angle MBC$ and $\angle PCB = \angle ADP$
$\implies \angle (AM,PD) = \angle (PC, BM)$

Let $F$ be a point such that $DPCF$ is a parallelogram.
Noticed that $DF \parallel PC \parallel AM \implies \angle EDF = \angle EAM$
Analogously, $\angle ECF = \angle EBM$.
Since $\angle EDF = \angle DAM = \angle MBC = \angle ECF$, by isogonality lemma, we have $\angle DEF = \angle PEC$.
Also, since $\angle EDF = \angle EAM = \angle MCD = \angle MDC$ and analogously, $\angle ECF = \angle MCD$,
we have $F$ and $M$ are isogonal conjugates and this implies $\angle DEF = \angle MEC$

Finally, $\angle PEC = \angle DEF = \angle MEC \implies E-M-P$. $\square$
https://i.imgur.com/uhHECAS.png
This post has been edited 1 time. Last edited by AlephG_64, Jul 17, 2024, 12:29 PM
Reason: remove att
Z K Y
The post below has been deleted. Click to close.
This post has been deleted. Click here to see post.
Mahdi_Mashayekhi
695 posts
#7
Y by
Let $CP$ and $DP$ meet $AD$ and $BC$ at $T,Q$. Note that $\angle PTD = \angle DPC - \angle PDT = \angle PDB = \angle PCA = \angle DPC - \angle PCQ = \angle PQC$ so $TDCQ$ is cyclic. Note that $\angle DAC = \angle ATC + \angle ACT = \angle DTP + \angle CQP$ so if circles $PDT$ and $PCQ$ meet at $K$ we have that $K$ lies on $ABCD$. Using Radical axis we have that $TD,QC,KP$ are concurrent to we need to prove $M$ lies on $KP$ which is true since $\angle DKM = \frac{\angle DAC}{2} = \angle DTP = \angle DKP$.
This post has been edited 1 time. Last edited by Mahdi_Mashayekhi, Jul 17, 2024, 3:59 PM
Z K Y
The post below has been deleted. Click to close.
This post has been deleted. Click here to see post.
ItzsleepyXD
145 posts
#8
Y by
easy to see by radical axis
Z K Y
The post below has been deleted. Click to close.
This post has been deleted. Click here to see post.
Marinchoo
407 posts
#9 • 1 Y
Y by isomoBela
Our aim is to show $M$ lies on the radical axis of the circumcircles of $\triangle PAD$ and $\triangle PBC$. This is sufficient as $P$ lies on both circles and $\overline{AD}\cap\overline{BC}$ lies on the radical axis by power of a point.

To compute the power of $M$ with respect to the two circumcircles, let lines $\overline{MD}$ and $\overline{MC}$ intersect the circumcircles of $\triangle PAD$ and $\triangle PBC$ at points $D'$ and $C'$, respectively. We want to show $MC\cdot MC' = MD \cdot MD'$, which is equivalent to $CC' = DD'$ as $MC = MD$ by definition. We go about computing $CC'$ and $DD'$ by using the trigonometric variant of Ptolemy's theorem for the cyclic $PADD'$ and $PBCC'$:
\begin{align*}
    DD'\sin\angle PDA &= DA \sin\angle PDD' - DP\sin\angle ADD'\\
    CC'\sin\angle PCB &= CB \sin\angle PCC' - PC \sin\angle BCC'.
\end{align*}Luckily, $\angle PDA = \angle PCB$, so now we just want to show
\[DA \sin \angle PDM - DP \sin\angle ABM = CB \sin\angle PCM - PC\sin\angle BAM.\]At this point, it's only logical that $\triangle PDC \sim \triangle MAB$. Indeed, assume it's not and let $P'$ be such that $\triangle DP'C \sim \triangle AMB$ and $P'$ lies in the same halfplane as $A$ and $D$ with respect to line $\overline{CD}$. Then clearly $\angle DP'C = \angle AMB = \angle DPC$, and furthermore:
\[\angle ADP' = \angle ADC - \angle P'DC = \angle ADC - \angle MAB\]\[\angle BCP' = \angle BCD - \angle P'CD = \angle BCD - \angle MBA\]As $\angle ADC - \angle MAB = \angle AMB - \angle MAC = \angle BCD - \angle MBA$, we get $\angle P'DA = \angle P'CB$. It's now obvious that $P \equiv P'$ as
\[\angle P'DA = \frac{1}{2}(\angle DP'C+\angle DXC) = \frac{1}{2}(\angle DPC + \angle DXC) = \angle PDA.\]We are now ready to prove the line from above as $\angle PDM = \angle DAB$, $\angle PCM = \angle CBA$, $DP = AM\cdot \frac{CD}{AB}$, and $CP = BM \cdot \frac{CD}{AB}$ from $\triangle DPC \sim \triangle AMB$. Denote $\angle DAC = x$, $\angle ADB = y$, $\angle CAB = z$, and $\angle DCA = t = \pi-x-y-z$. Then
\begin{align*}
    DA \sin \angle PDM - DP \sin\angle ABM &= DA \sin(x+z) - AM \cdot \frac{CD}{AB}\sin (x/2+y+z)\\
    &= \frac{R}{\sin y}\left(\sin (x+y+z) \sin(x+z) \sin y - \sin^2(x/2+y+z) \sin x\right)\\
    &=\frac{R}{4\sin y} (\sin(2x + 2z) - 2 \sin(x) - \sin(2y + 2z) + \sin(2y))
\end{align*}where $R$ is the radius of the circumcircle of $ABCD$. Analogously,
\begin{align*}
    CB\sin\angle PCM - PC\sin\angle BAM &= CB\sin (y+z) - BM\cdot\frac{CD}{AB} \sin(x/2+z)\\
    &=\frac{R}{\sin y}\left(\sin z \sin(y+z)  \sin y- \sin^2(x/2+z)\sin x\right)\\
    &=\frac{R}{4\sin y} (\sin(2x+2z) - 2\sin(x) - \sin(2y+2z)+\sin(2y)).
\end{align*}With this, the solution is complete.
Z K Y
The post below has been deleted. Click to close.
This post has been deleted. Click here to see post.
blueberryfaygo_55
340 posts
#10 • 2 Y
Y by megarnie, centslordm
Claim. $DP \parallel BM$ and $CP \parallel AM$.
Proof. Let $E = AC \cap BD$. We have \begin{align*}
\angle ADB &= \angle CPD \\
&= 180^{\circ} - \angle PDB - \angle PCA - \angle BDC - \angle ACD \\
&= 180^{\circ} - 2 \angle PDB - \angle AED \\
&= \angle AEB - 2 \angle PDB.
\end{align*}This means that \begin{align*}
2 \angle PDB &= \angle AEB - \angle ADB \\
&= \angle DAE \\
&= \angle DAC \\
&= \angle DBC.
\end{align*}However, by the Incenter-Excenter Lemma, $BM$ is the angle bisector of $\angle DBC$, so $2 \angle DBM = \angle DBC$, which gives $\angle PDB = \angle DBM$, implying that $PD \parallel BM$. Analogously, we obtain $PC \parallel AM$, as desired. $\blacksquare$

Now, let $PD$ meet $\omega$ again at $E$, $PC$ meet $\omega$ again at $F$. It follows that $DEBM$ and $AMCF$ are isoceles trapezoids and $BE = DM = MC = AF$.

Claim. $ADMF$ and $BCME$ are isoceles trapezoids.
Proof. Since $\angle DAM$ and $\angle AMF$ are half the lengths of equal arcs, they must equal. Thus, $AD \parallel MF$, but $ADMF$ is cyclic, so $ADMF$ is an isoceles trapezoid. Similarly, $\angle MEC = \angle ECB$, so $BCME$ is also an isoceles trapezoid and $BC \parallel ME$. $\blacksquare$

Let $AF$ and $BE$ meet at $X$, $AD$ and $BC$ meet at $T$.

Claim. $M$, $P$, $X$ are collinear.
Proof. Since $FP \parallel AM$, $PE \parallel MB$, and $FE \parallel AB$, $\Delta FPE$ and $\Delta AMB$ are homothetic. Thus, $AF$, $BE$, $MP$ concur at the center of homothety $X$, giving the claim. $\blacksquare$

Claim. $M$, $T$, $X$ are collinear.
Proof. $FME$ and $ATB$ are homothetic triangles, so $TM$, $AF$, $EB$ concur at the center of homothety $X$ similar to above. $\blacksquare$

The two claims above imply that $T,M,P$ are collinear, which is exactly the desired condition, so we are done. $\blacksquare$
Z K Y
The post below has been deleted. Click to close.
This post has been deleted. Click here to see post.
Miquel-point
479 posts
#11 • 1 Y
Y by JollyEggsBanana
This was P1 on the first Romanian IMO TST this year.
I solved it with homothety and Pappus's theorem - will post a full write up later.
Z K Y
The post below has been deleted. Click to close.
This post has been deleted. Click here to see post.
Strudan_Borisov
16 posts
#12
Y by
Let $AD\cap BC = E, CP\cap AB =X, DP\cap AB = Y$. We'll prove $\overline{EMP}$ is the radical axis $\rho$ of $\odot ADY$ and $\odot BCX$. The identity $ED.EA=EC.EB$ means $E\in \rho$. A simple angle chase gives that $\angle MDP = \angle DAY$ which means $MD$ is the tangent to $\odot ADY$ at $D$. Therefore, $\operatorname{Pow}_{\odot ADY}(M)=MD^2$; similarly $\operatorname{Pow}_{\odot CBX}(M) = MC^2$. Since $MC=MD$, $M\in\rho$. Notice that \[\angle YXC = \angle BXC = 180^{\circ}-\angle CBA-\angle PCB=\angle ADC - \angle ADP = \angle PDC = \angle YDC,\]meaning $XYCD$ is cyclic. Then $PX.PC=PY.PD\implies P\in\rho$, which finishes the solution.
This post has been edited 1 time. Last edited by Strudan_Borisov, Jul 18, 2024, 6:07 AM
Z K Y
The post below has been deleted. Click to close.
This post has been deleted. Click here to see post.
EpicBird08
1753 posts
#13 • 1 Y
Y by centslordm
I spent 2 hours on this problem, WITH GEOGEBRA.

We first remark that $P$ is unique, since $\angle ADP = \angle PCB$ must vary until we get that the angle between the two lines is $\angle BDA.$

Now given this, we will actually classify $P.$ Let the line through $D$ parallel to $BM$ and the line through $C$ parallel to $AM$ intersect at point $P'.$ We claim that $P' = P.$

First of all, by the parallel lines, it is clear that $\angle CP'D = \angle BMA = \angle BDA.$ Second, we see that $$\angle ADP' = \angle(AD, BM) = \angle ADB - \angle MAD = \angle ADB - \angle MAC = \angle(BC, AM) = \angle P'CB,$$so $P'$ satisfies the problem conditions and thus is equal to $P.$

Now we will show that $$\frac{[BPC]}{[APD]} = \frac{[BMC]}{[AMD]}.$$This finishes, since that will readily imply that $PM$ passes through $AD \cap BC,$ at which point we are done.

We first compute the first ratio. The area on top is equal to $\frac{BC \cdot CP \cdot \sin \angle BCP}{2}$ while the area on bottom is equal to $\frac{AD \cdot DB \cdot \sin \angle ADP}{2}.$ Dividing the two and noting that $\angle ADP = \angle BCP,$ we get $$\frac{[BPC]}{[APD]} = \frac{BC \cdot CP}{AD \cdot DP}.$$Now we compute the second ratio. The area on top is equal to $\frac{BC \cdot BM \cdot \sin \angle CBM}{2}$ while the area on bottom is equal to $\frac{AD \cdot AM \cdot \sin \angle DAM}{2}.$ Dividing the two and noting that $\angle DAM = \angle MBC,$ we get $$\frac{[BMC]}{[AMD]} = \frac{BC \cdot BM}{AD \cdot AM}.$$It thus suffices to show that $\frac{PC}{PD} = \frac{BM}{AM}.$ In fact, $\angle CPD = \angle AMD$, and if $DP$ hits $(ABCD)$ again at point $Z,$ then $MD = ZB = MC,$ so $CB \parallel ZM$ and so $\angle PDC = \angle BAM.$ Thus $\triangle CPD \sim \triangle BMA,$ giving the desired length ratio, and we are done.
This post has been edited 1 time. Last edited by EpicBird08, Jul 17, 2024, 11:18 PM
Z K Y
The post below has been deleted. Click to close.
This post has been deleted. Click here to see post.
foolish07
24 posts
#14
Y by
Let $BC\cap AD=E, BC\cap DP=Y, AD\cap CP=X.$
Then clearly $D,X,Y,C$ is cyclic and $AB \parallel XY$
And Let $M'$ is $AM' \parallel XP$, $BM' \parallel YP$
Then $\angle M'AC = \angle ACX = \angle ACY - \angle PCY = \angle DPC-\angle PCY =\angle DYC =\angle DXC =\angle DAM'$
Similarly $\angle M'BD=\angle CBM'$
Thus $M'=M$
Thus $\Delta MAB$ and $\Delta PXY$ is homothetic, and their homotheric center is $E \implies E-M-P$
Z K Y
The post below has been deleted. Click to close.
This post has been deleted. Click here to see post.
pi_quadrat_sechstel
601 posts
#15
Y by
GrantStar wrote:
Let $ABCD$ be a cyclic quadrilateral with $\angle BAD < \angle ADC$. Let $M$ be the midpoint of the arc $CD$ not containing $A$. Suppose there is a point $P$ inside $ABCD$ such that $\angle ADB = \angle CPD$ and $\angle ADP = \angle PCB$.

Prove that lines $AD, PM$, and $BC$ are concurrent.

Let $\Omega$ be the circumcircle of $ABCD$ and let $A'=CP\cap AD,B'=NC\cap DP,X=BC\cap AD$ and $Y=A'B'\cap CD$. We have $\angle A'CB'=\angle PCB=\angle ADP=\angle A'DB'$. So $A',B',C$ and $D$ lie on a circle $\Gamma$. Let $O$ be the center of $\Gamma$. By angle chasing we get $\angle BB'D=\angle B'DB$ and thus $\angle COD=2\angle CB'D=\angle CBD$. So $|BB'|=|BD|$ and analogously $|AA'|=|AC|$ and $O$ lies on $\Omega$.

Now $O$ and $M$ are two points on $\Omega$ and the perpendicular bisector of $CD$. We have $BO\perp DP$ since B and $O$ lie on the perpendicular bisector of $B'D$. We have $BM\parallel DP$ since $\angle PDB=\frac{1}{2}\angle CBD=\angle MBD$. So $O\neq M$ and $OM$ is a diameter of $\Omega$.

By Thales we have $\angle MDO=\angle OCM=90^\circ$. So $MC$ and $MD$ are tangent to $\Gamma$. So $CD$ is the polar of $M$ w.r.t. $\Gamma$. By pole-polar duality $M$ lies on the polar of $Y$ w.r.t. $\Gamma$. By Brocard $PX$ is the polar of $Y$ w.r.t. $\Gamma$. Thus $BC,AD$ and $PM$ concur at $X$.
Z K Y
The post below has been deleted. Click to close.
This post has been deleted. Click here to see post.
KST2003
173 posts
#16
Y by
Let $CP$ and $DP$ meet $(ABC)$ at $C'$ and $D'$. Then since $\angle ADP = \angle PCB$, $D'C'AB$ is an isosceles trapezoid with $D'C' \parallel AB$. Now let $M'$ be the point such that $AM' \parallel CP$ and $BM' \parallel PD$. Then since $\angle AM'B = \angle CPD = \angle ADB$, $M'$ lies on $(ABC)$, and we also have $M'C = C'A = D'B = M'D$, so $M' = M$. Since $\triangle ABM$ and $\triangle C'D'P$ are homothetic, we see that $\overline{D'B}$, $\overline{C'A}$ and $\overline{PM}$ are concurrent at a point $Z$. Now Pascal's theorem on the hexagon $BD'DAC'C$ shows that $Z$, $P$ and $\overline{AD} \cap \overline{BC}$ are collinear so we are done.
Z K Y
The post below has been deleted. Click to close.
This post has been deleted. Click here to see post.
Ianis
412 posts
#17
Y by
The angle conditions give $CP\parallel AM$ and $DP\parallel BM$ (this has already been proved in previous solutions, so I won't write it). Now let $A'=AM\cap CD$, $B'=BM\cap CD$ and $G=PM\cap CD$. Then by Thales/homothety we get that$$GA'\cdot GD=GB'\cdot GC,$$so $G$ is on the radical axis of $(MA'D)$ and $(MB'C)$. Finally, the inversion with centre $M$ and radius $MC=MD$ maps $AD$ to $(MA'D)$ and $BC$ to $(MB'C)$, which means that $AD\cap BC$ also lies on the radical axis of $(MA'D)$ and $(MB'C)$. Done.
Z K Y
The post below has been deleted. Click to close.
This post has been deleted. Click here to see post.
jrpartty
44 posts
#18
Y by
Let $CP$ and $DP$ meet the circumcircle of $\square ABCD$ again at $Q,R$, respectively. Denote, $X,Y$ the intersections of $AB,CD$ and $AQ,BR$, respectively.

By several angle chasing, we obtain that $MR\parallel BC$, $MQ\parallel AD$ and $QR\parallel AB$. Hence, $\triangle MQR$ and $\triangle XAB$ are homothetic, implying $Y,M,X$ are collinear.

Applying Pascal to $ADRBCQ$, we obtain that $Y,P,X$ are also collinear, so we are done. $\square$
This post has been edited 1 time. Last edited by jrpartty, Jul 19, 2024, 2:33 PM
Z K Y
The post below has been deleted. Click to close.
This post has been deleted. Click here to see post.
Shreyasharma
682 posts
#19
Y by
The angle conditions imply that $AM \parallel PC$ and $BM \parallel PD$. Define $X = BC \cap DP$, $Y = AD \cap CP$ and $Z = PM \cap (ABCD)$. By construction $CPXZ$, $DPYZ$ and $CDXY$ are all cyclic (!) so that radical axis finishes.
Z K Y
The post below has been deleted. Click to close.
This post has been deleted. Click here to see post.
sami1618
909 posts
#20
Y by
Let $CP$ meet $AD$ at $E$ and $DP$ meet $BC$ at $F$. As $\angle PDA=\angle PCB$, $EFCD$ is cyclic. We claim that $MC$ is tangent to $(EFCD)$ $$\angle MCD=\frac{1}{4}\overarc{CD}=\frac{1}{2}\overarc{AB}-\frac{1}{2}(\angle C+\angle D+\frac{1}{2}\overarc{AB}-180^{\circ})=\angle DPC-\angle ADP=\angle DEC$$Similarly, $MD$ is tangent to $(EFCD)$. We are done by Pascal's Theorem on $CCEDDF$.
Z K Y
The post below has been deleted. Click to close.
This post has been deleted. Click here to see post.
hectorleo123
344 posts
#21 • 1 Y
Y by BorivojeGuzic123
GrantStar wrote:
Let $ABCD$ be a cyclic quadrilateral with $\angle BAD < \angle ADC$. Let $M$ be the midpoint of the arc $CD$ not containing $A$. Suppose there is a point $P$ inside $ABCD$ such that $\angle ADB = \angle CPD$ and $\angle ADP = \angle PCB$.

Prove that lines $AD, PM$, and $BC$ are concurrent.
I have 2 short solutions :yoda:
Solution 1
Solution 2
This post has been edited 3 times. Last edited by hectorleo123, Jul 22, 2024, 3:51 PM
Z K Y
The post below has been deleted. Click to close.
This post has been deleted. Click here to see post.
VicKmath7
1389 posts
#22
Y by
Solution
Z K Y
The post below has been deleted. Click to close.
This post has been deleted. Click here to see post.
SerdarBozdag
892 posts
#23
Y by
Gorgeous :first:

Define $Q = AD \cap BC$, $L = PD \cap (ABCD)$, $K = PC \cap (ABCD)$ and $J = AK \cap BL$. Let $\angle PDA = \angle PCB = \beta$ and $\angle PDB = \angle PCA = \alpha$. Because $\angle CPD = \alpha + \beta$ and $\angle ADP + \angle BCP = \angle CPD + \angle CQD \implies \angle DQC = \beta - \alpha$. Additionally, $\angle DBC = \angle ADB - \angle DQC \implies 2 \alpha$. This shows $MK \parallel AD$, $ML \parallel BC$ and $KL \parallel AB$ due to the fact that $\widehat{AK} = \widehat{BL} = \widehat{MC} = \widehat{MD}$.

$J$ is the homothety center of $KLM$ and $ABQ$. This shows that $M \in JQ$. By Pascal Theorem on $KADLBC$, $P \in JQ$ which is sufficient.
Z K Y
The post below has been deleted. Click to close.
This post has been deleted. Click here to see post.
SHZhang
109 posts
#24
Y by
Let $X = AD \cap CP$ and $Y = BC \cap DP$. Then $\angle XCY = \angle PCB = \angle ADP = \angle XDY$ gives $(XDCY)$ cyclic. Let $Q = AC \cap DP$ and $R = BD \cap CP$. Then $\angle QDR = \angle ADB - \angle ADP = \angle ACB - \angle PCB = \angle QCR$ gives $(QRCD)$ cyclic. Since \[ \angle DYB = \angle QYC = 180^\circ - \angle YQC - \angle QCY = 180^\circ - \angle YQC - \angle ADB = 180^\circ - \angle YQC - \angle CPD = \angle QCR = \angle QDR = \angle YDB, \]we have $BD = BY$, and similarly $AC = AX$. Then $\angle DBM = \frac12 \angle DBC = \frac12 (180^\circ - \angle DBY) = \angle YDB$, so $BM \parallel DY$ and similarly $AM \parallel CX$.

Let $Z$ be the second intersection of $PM$ with $(ABCD)$. Then $\angle XPZ = \angle AMZ = \angle ADZ = \angle XDZ$ gives $(XDPZ)$ cyclic, and similarly $(YCPZ)$ is cyclic. The desired claim follows from radical axis on $(XDPZ)$, $(YCPZ)$, and $(XDCY)$.
Z K Y
The post below has been deleted. Click to close.
This post has been deleted. Click here to see post.
keglesnit
176 posts
#25
Y by
Solution using the isogonal conjugate of $P$ in triangle $CDE$ where $E=AD\cap BC$.

Let $Q$ be the isogonal conjugate. Then
$$\angle CPD=180^\circ-\angle PCD-\angle PDC=\angle EDC-\angle ECD-180^\circ+2\angle QDC=2\angle QDC-\angle CED.$$As $\angle ADB=\angle E+\angle CBD$, we find $\angle E+\angle DBM=\angle QDC$. Now, let the perpendicular bisector of $CD$ intersect $(ABCD)$ at $N\neq M$ and $(CDE)$ at $U$ and $V$ with $U$ in the minor arc $CD$. Then
$$\angle QDM=\angle QDC+\angle MDC=\angle E+\angle DBM+\angle CBM=\angle CED+\angle CND.$$Thus we find $\angle QDM=180^\circ-\angle DMC+180^\circ-\angle DUC=2\angle UDM$. I.e. $DU$ bisects $\angle MDQ$. Appolonius circle gives that $EU$ bisects $\angle EMQ$, and therefore $EM$ and $EQ$ are isogonal. Hence $EM$ pass through $P$ by construction of $Q$.
This post has been edited 1 time. Last edited by keglesnit, Jul 22, 2024, 9:45 AM
Z K Y
The post below has been deleted. Click to close.
This post has been deleted. Click here to see post.
AngeloChu
471 posts
#26
Y by
let $PM$ intersect $(ABCD)$ again at point $P'$, and let $AD$ intersect $PC$ at $A'$, and similarly define $B'$
since $ADP = PCB$, we have that $A'B'CD$ are concyclic, so we can just angle chase for $DB' \parallel BM$ and $CA' \parallel AM$
then, $PA'D=DAM=DP'P$ and $CB'D=CBM=CP'M$, so $A'DPP'$ and $B'CPP'$ are both cyclic
thus, $PM$ is a radical axis of these two circles and since $A'B'CD$ is also cyclic, we have that $AD$, $PM$, and $BC$ are concurrent on $PM$
Z K Y
The post below has been deleted. Click to close.
This post has been deleted. Click here to see post.
Anancibedih
18 posts
#27
Y by
Let $T$ be the intersection of the line passing through $P$ and parallel to $BC$ with $BD$. Since $\angle{BCP}=\angle{CPT}$, then $\angle{TPD}=\angle{TDP}$. $\angle{BTP}=2\angle{TDP}$. It is also equal to $\angle{CBD}$. So $\angle{CBD}=2\angle{TPD}$. Since point $M$ is the midpoint, $\angle{MPD}=\angle{BDP}$. It can be seen that $MB||DP$ Since $\angle{BMA}=\angle{BDA}=\angle{CPD}$ it is $CP||MA$. Now we have to prove that $CR||MA$ for the point $R$, which is the intersection of the line passing through $D$ and parallel to $MB$ with the line passing through $BC\cap AD$ and $M$. Let $BC\cap AD=Z$. Let $AM\cap ZB=J$ and $BM\cap ZA=Q$. Since $\angle{CBM}=\angle{MAD}$, then $\angle{BQA}=\angle{BJA}$. $JQAB$ is cyclic. Thus $JQ||CD$. $\frac{|ZJ|}{|ZC|}=\frac{|ZQ|}{|ZD|}$. Also with $MQ||DR$, by the similarity in the triangle $ZDR$, $\frac{|ZQ|}{|ZD|}=\frac{|ZM|}{|ZR|}$. Since $\frac{|ZJ|}{|ZC|}=\frac{|ZQ|}{|ZD|}$ as we just found, $\frac{|ZJ|}{|ZC|}=\frac{|ZM |}{|ZR|}$ becomes. Thus, $JM||CR$ in the triangle $ZCR$. The proof ends.
This post has been edited 2 times. Last edited by Anancibedih, Sep 17, 2024, 6:22 PM
Z K Y
The post below has been deleted. Click to close.
This post has been deleted. Click here to see post.
SomeonesPenguin
128 posts
#28 • 2 Y
Y by ehuseyinyigit, zzSpartan
Rather quick trig sol that I found.

Let circle $(DCP)$ intersect $AD$ and $BC$ at $F$ and $G$ respectively and let $AD$ intersect $BC$ at $N$. Let the arcs $DC$, $AB$, $AD$ and $BC$ be equal to $4a$, $2b$, $2c$ and $2d$ respectively. We have that $FG$ is parallel to $AB$ and $P$ is the midpoint of arc $FG$. Quick angle chasing in $(DCGPF)$ gives $\angle PDC = d + a$ and $\angle PCD = a + c$. Also note that $\angle NDP = \angle NCP$. We apply trig ceva in $\triangle{NDC}$ with point $P$ $$\frac{\sin(\angle DNP)}{\sin(\angle PNC)}\cdot\frac{\sin(\angle NCP)}{\sin(\angle PCD)}\cdot\frac{\sin(\angle PDC)}{\sin(\angle PDN)}=1$$
So we get that: $$\frac{\sin(\angle DNP)}{\sin(\angle PNC)} = \frac{\sin(a+c)}{\sin(a+d)}$$
Another quick angle chase gives $\angle MDN = a+c$, $\angle MCN = a+d$ and $\angle DCM = \angle CDM$. So from trig ceva in $\triangle{DCN}$ with point $M$ we get: $$\frac{\sin(\angle DNM)}{\sin(\angle MNC)}\cdot\frac{\sin(\angle NCM)}{\sin(\angle MCD)}\cdot\frac{\sin(\angle CDM)}{\sin(\angle MDN)} = 1$$
So we get that: $$\frac{\sin(\angle DNM)}{\sin(\angle MNC)} = \frac{\sin(a+c)}{\sin(a+d)} = \frac{\sin(\angle DNP)}{\sin(\angle PNC)}$$
And since $\angle DNM + \angle MNC = \angle DNP + \angle PNC = \angle DNC$ we get that $\angle DNM = \angle DNP$ and $\angle MNC = \angle PNC$ so $N$, $M$ and $P$ are collinear. Done!
This post has been edited 1 time. Last edited by SomeonesPenguin, Aug 1, 2024, 7:32 PM
Z K Y
The post below has been deleted. Click to close.
This post has been deleted. Click here to see post.
fearsum_fyz
52 posts
#29 • 2 Y
Y by sami1618, OronSH
Let $\overrightarrow{\rm CP}$ and $\overrightarrow{\rm DP}$ meet $\overleftrightarrow{\rm AD}$ and $\overleftrightarrow{\rm BC}$ at $X$ and $Y$.

Claim: $\Delta{AMB}$ and $\Delta{XPY}$ are homothetic.
Proof. Angle chasing.

We are done.
Attachments:
Z K Y
The post below has been deleted. Click to close.
This post has been deleted. Click here to see post.
OronSH
1747 posts
#30 • 5 Y
Y by GrantStar, ihatemath123, Zhaom, ehuseyinyigit, MS_asdfgzxcvb
Let $CP\cap\omega,DP\cap\omega,AD\cap BC=X,Y,Z$. Angle chase to get $ZAMB,MXPY$ homothetic which finishes.
Z K Y
The post below has been deleted. Click to close.
This post has been deleted. Click here to see post.
Zehra19
6 posts
#31 • 2 Y
Y by ehuseyinyigit, Umudlu
Let $AD\cap BC=T$$\quad$ $\angle BDA=\angle BCA$ and $\angle PCB=\angle PDA$ then $\angle PCA=\angle PDB$

$\angle CPD+\angle PDC+\angle PCD=180^\circ=\angle BDA+ \angle BAD+\angle DBA$

$\angle DBA+ \angle DAB=\angle PCD+ \angle PDC$

$\implies$ $2\angle PDB=\angle CAD=2\angle CDM$ $\quad$ so, $\quad$ $\angle CDM=\angle PDB$
$\implies$ $\angle PDC=\angle BDM=\angle MCT$ $\quad$and analogously$\quad$ $\angle PCD=\angle MDT$

$$\frac{MT}{\sin \angle MCT}=\frac{MC}{\sin \angle MTC}\quad and \quad \frac{MT}{\sin \angle MDT}=\frac{MD}{\sin \angle MTD}$$
$$\frac{PT}{\sin \angle PCT}=\frac{PC}{\sin \angle PTC} \quad and \quad \frac{PT}{\sin \angle PDT}=\frac{PD}{\sin \angle PTD}$$
$$\frac{\sin\angle MDT}{\sin\angle MCT}=\frac{\sin\angle MTD}{\sin\angle MTC}\quad and \quad 1=\frac{\sin\angle PDT}{\sin\angle PCT}=\frac{\sin\angle PTD}{\sin\angle PTC}\cdot\frac{PC}{PD}$$
$$\frac{\sin\angle PTC}{\sin\angle PTD}=\frac{PC}{PD}=\frac{\sin\angle PDC}{\sin\angle PCD}=\frac{\sin\angle MCT}{\sin\angle MDT}=\frac{\sin\angle MTC}{\sin\angle MTD}$$$$\implies \frac{\sin\angle PTC}{\sin\angle PTD}=\frac{\sin\angle MTC}{\sin\angle MTD}$$
Let $\quad$ $\angle CTD=\alpha$, $\quad$ $\angle PTC=\beta$, $\quad \angle MTC=\theta$

$$\frac{\sin\angle \beta}{\sin\angle (\alpha-\beta)}=\frac{\sin\angle \theta}{\sin\angle (\alpha-\theta)}$$$$\implies \beta=\theta$$$$\angle PTC=\angle MTC $$$\implies$ $P,T,M$ are colleniar. Done!
Z K Y
The post below has been deleted. Click to close.
This post has been deleted. Click here to see post.
Ege_Saribass
29 posts
#32 • 1 Y
Y by Anancibedih
Let $AD \cap BC = \{T\}$. Let the second intersections of $PC$ and $PD$ with $(ABCD)$ be $X$ and $Y$, respectively. We are going to show $\overline{TMP}$.
By easy angle chasing, we conclude $\angle PDB = \angle PCA \implies arc(AX) = arc(YB)$.
By angles, also we get $arc(AX) + arc(XY) + arc(YB) = arc(XY) + arc(DC) \implies arc(DM) = arc(MC) = arc(AX) = arc(YB)$
Hence, $XY || AB$, $XM || AD$ and $YM || BC$. So we conclude that $\triangle{ATB}$ and $\triangle{XMY}$ are homothetic.
Thus, $AX$, $BY$ and $TM$ are concurrent. Let $Z$ be the intersection point of these lines. So we know that $\overline{TMZ}$.
Now Pascal on $(ADYBCX)$ gives us $\overline{TPZ}$.
$\overline{TMZ}$ and $\overline{TPZ}$ implies $\overline{TMP}$.
$\blacksquare$
Attachments:
This post has been edited 2 times. Last edited by Ege_Saribass, Sep 14, 2024, 3:44 PM
Reason: colour:)
Z K Y
The post below has been deleted. Click to close.
This post has been deleted. Click here to see post.
kamatadu
480 posts
#33 • 1 Y
Y by SilverBlaze_SY
Solved with SilverBlaze_SY. We give two solutions, one with MMP (mine) and the other one by chasing angles (SilverBlaze's).
[asy]
/* Converted from GeoGebra by User:Azjps using Evan's magic cleaner https://github.com/vEnhance/dotfiles/blob/main/py-scripts/export-ggb-clean-asy.py */ /* A few re-additions are done using bubu-asy.py. This adds the dps, xmin, linewidth, fontsize and directions. https://github.com/Bubu-Droid/dotfiles/blob/master/bubu-scripts/bubu-asy.py */ pair A = (7.77952,50.31114); pair B = (-7.41116,-7.83103); pair C = (41.85666,-8.11885); pair D = (51.58016,23.65726); pair M = (50.74270,6.53777); pair P = (18.65367,15.52170); pair F = (104.39862,-8.48421); pair X = (37.38301,20.14939); pair Y = (32.01336,1.91008);
import graph; size(15cm); pen dps = linewidth(0.5) + fontsize(13); defaultpen(dps); real xmin = -5, xmax = 5, ymin = -5, ymax = 5;
pen ffxfqq = rgb(1.,0.49803,0.); draw(A--B, linewidth(0.6)); draw(B--C, linewidth(0.6)); draw(C--A, linewidth(0.6)); draw(circle((17.36719,16.75068), 34.90312), linewidth(0.6)); draw(A--D, linewidth(0.6)); draw(D--M, linewidth(0.6)); draw(M--C, linewidth(0.6)); draw(B--M, linewidth(0.6) + ffxfqq); draw((24.59535,0.07721)--(23.56486,-2.04245), linewidth(0.6) + ffxfqq); draw((24.59535,0.07721)--(22.69624,1.47304), linewidth(0.6) + ffxfqq); draw((21.66576,-0.64662)--(20.63528,-2.76630), linewidth(0.6) + ffxfqq); draw((21.66576,-0.64662)--(19.76666,0.74919), linewidth(0.6) + ffxfqq); draw(D--F, linewidth(0.6) + linetype("4 4") + red); draw(F--C, linewidth(0.6) + linetype("4 4") + red); draw(P--F, linewidth(0.6) + linetype("4 4") + red); draw(P--D, linewidth(0.6) + ffxfqq); draw((38.04650,20.31333)--(37.01602,18.19365), linewidth(0.6) + ffxfqq); draw((38.04650,20.31333)--(36.14740,21.70915), linewidth(0.6) + ffxfqq); draw((35.11691,19.58948)--(34.08643,17.46980), linewidth(0.6) + ffxfqq); draw((35.11691,19.58948)--(33.21781,20.98531), linewidth(0.6) + ffxfqq); draw(P--C, linewidth(0.6) + blue); draw((32.36897,1.54776)--(30.01987,1.35631), linewidth(0.6) + blue); draw((32.36897,1.54776)--(32.60426,3.89287), linewidth(0.6) + blue); draw((30.25517,3.70142)--(27.90607,3.50997), linewidth(0.6) + blue); draw((30.25517,3.70142)--(30.49046,6.04653), linewidth(0.6) + blue); draw(A--M, linewidth(0.6) + blue); draw((31.37491,26.27079)--(29.02581,26.07934), linewidth(0.6) + blue); draw((31.37491,26.27079)--(31.61021,28.61591), linewidth(0.6) + blue); draw((29.26111,28.42446)--(26.91201,28.23301), linewidth(0.6) + blue); draw((29.26111,28.42446)--(29.49641,30.76957), linewidth(0.6) + blue);
dot("$A$", A, NW); dot("$B$", B, SW); dot("$C$", C, SE); dot("$D$", D, NE); dot("$M$", M, NE); dot("$P$", P, NW); dot("$F$", F, NE); dot("$X$", X, N); dot("$Y$", Y, dir(270));  [/asy]

The crux of the problem is to note that $MA\parallel CP$ and $MB\parallel DP$.

Claim: $MA\parallel CP$ and $MB \parallel DP$.
Proof. Let $X=MA\cap DP$ and $Y=MB\cap CP$. Firstly, note that, \[ \measuredangle YPX=\measuredangle CPD=\measuredangle ADB =\measuredangle AMB = \measuredangle XMY .\]Now note that, \begin{align*} \measuredangle PXM=\measuredangle DXA &= \measuredangle DAX+\measuredangle XDA\\ &= \measuredangle DAM+\measuredangle PDA\\ &=\measuredangle MBC+\measuredangle BCP\\ &=\measuredangle YBC +\measuredangle BCY\\ &=\measuredangle BYC=\measuredangle MYP .\end{align*}Now since the opposite angles of the quadrilateral $PYMX$ are equal, we must have that $PYMX$ is a parallelogram. This gives us our desired result. $\blacksquare$

Now we redefine $P$ as the intersection of line through $D$ parallel to $MB$ with the line through $C$ parallel to $MA$.

This our problem statement can now be generalized as follows.
Quote:
Let $ABC$ be a triangle and let $M$ denote the midpoint of arc $\widehat{BC}$ not containing $A$. Let $D$ be any arbitrary point on $\odot(ABC)$. Let the line through $D$ parallel to $MB$ be $\ell_1$. Similarly, let the line through $C$ parallel to $MA$ be $\ell_2$. Define $P=\ell_1\cap \ell_2$. Prove that $AD$, $PM$ and $BC$ are concurrent.

Fix $A$, $C$ and $D$. We animate $B$ projectively on $\odot(ACD)$. Firstly note that $M$ has degree $0$ (as it is also fixed). So, the line $MA$ is fixed and has degree $0$. Thus $\ell_2$ also has degree $0$. Furthermore note that the line at infinity is also fixed as $\infty_{AC}$ and $\infty_{AD}$ are fixed.
Now note that as $MA$ is fixed, $CP$ is also fixed.
Then we have, \[ B\mapsto MB\mapsto MB\cap \ell_{\infty}\mapsto D\infty_{MB}\mapsto D\infty_{MB}\cap CP \text{ via }\odot(ACD)\mapsto \mathcal{P}(M) \mapsto \ell_{\infty}\mapsto \mathcal{P}(D) \mapsto CP \]is projective. This means that $P$ has degree $1$. So the line $PM$ has degree $1+0=1$
Now note that, \[ B\mapsto CB\text{ via }\odot(ACD)\mapsto \mathcal{P}(C) \]is projective. This means that the line $CB$ has degree $1$. Thus finally, the condition that $AD$, $CB$ and $PM$ has degree $0+1+1=2$. Firstly note that when $B=C$, we can reduce the degree by $1$ using Zack's Lemma.
So now we need to check for just two cases.
  • $B=D$ -- Then note that $P=CP\cap DM$. So, $AD\cap PM\cap BC=D$.
  • $B=A$ -- Then note that $DP\parallel AM\parallel CP$ which implies that $P\equiv \infty_{AM}$. Then clearly $AD\cap PM\cap BC=A$.
And we are done.
This post has been edited 2 times. Last edited by kamatadu, Sep 27, 2024, 2:59 PM
Z K Y
The post below has been deleted. Click to close.
This post has been deleted. Click here to see post.
bjump
1028 posts
#34
Y by
oronsh wrote:
2023 g3 is good warmup geo
Let $CP$, and $DP$ intersect $(ABCD)$ again at $C'$, and $D'$ respectively. Let $AD \cap BC = E$ . Let $BD' \cap AC' =X$. Let $C' M \cap DD'$, and $D'M \cap CC'$ be $F$ and $G$ respectively.
$\angle ADP = \angle PCB$ implies $D'C' \parallel AB$ and arcs $BD'$, $AC'$, $DM$, and $CM$ have equal length. Therefore $ADMC'$ is an isosceles trapezoid along with $BCMD'$. Observe by pascals on $DD'BCC'A$, $X$, $P$, and $E$ must be collinear. By pascals on $CC'MMDD'$ $FG \parallel CD$. Therefore $(FGC'D')$ is cyclic. Observe that $\angle D'FC' = \angle C'FD = 180^\circ - \angle D'DA= \angle D'C'X$ therefore $XC'$ is tangent to $(GFC'D')$. Now Pascals on $C'FD'D'GC'$ gives $X$, $P$, $M$ collinear. This combined with $X$, $P$, $E$ collinear gives us the desired result.
Z K Y
The post below has been deleted. Click to close.
This post has been deleted. Click here to see post.
HamstPan38825
8866 posts
#35 • 1 Y
Y by ihatemath123
Moral of the story: if you don't know what to do, apply Pascal and see what happens.

We let $\overline{CP}$ intersect $\overline{AD}$ at $E$ and $\overline{DP}$ intersect $\overline{BC}$ at $F$. The key claim is the following:

Claim: $BF=BD$ and $AC=AE$.

Proof: We instead imagine picking points $E'$ and $F'$ on $\overline{AD}$ and $\overline{BC}$ satisfying the length conditions, then show that $P = \overline{CE'} \cap \overline{DF'}$ satisfies both angle conditions; obviously it satisfies the latter one as $FEDC$ is cyclic (see below).

To see the former condition, $\angle CE'D = \frac 12 \angle CAD = \frac 12 \angle CBD = \angle CF'D$, so triangles $ACE'$ and $BDF'$ are similar. So \[\measuredangle CPD = \measuredangle(\overline{E'C}, \overline{F'D}) = \measuredangle(\overline{AE'}, \overline{DB}) = \measuredangle ADB\]as needed. $\blacksquare$

Now, note that the tangents at $C$ and $D$ to $(FEDC)$ meet at $M$ as $\angle MDC = \frac 12 \angle DAC = \angle DEC$. Thus by Pascal on $CEDDFC$ we have the desired concurrence.

Edit: after writing this solution I realized that triangles $PEF$ and $MAB$ are homothetic, which just like solves the problem. Geometry is too hard.
This post has been edited 1 time. Last edited by HamstPan38825, Dec 13, 2024, 4:39 AM
Z K Y
The post below has been deleted. Click to close.
This post has been deleted. Click here to see post.
lelouchvigeo
182 posts
#36 • 2 Y
Y by L13832, alexanderhamilton124
ratio spam
Z K Y
The post below has been deleted. Click to close.
This post has been deleted. Click here to see post.
peace09
5419 posts
#37 • 5 Y
Y by OronSH, centslordm, ihatemath123, imagien_bad, ehuseyinyigit
Tetrahedral Hyperprism :)
https://cdn.artofproblemsolving.com/attachments/9/3/1caac3b4764599760cbee386a69b30202b9cd9.png
Let $A@B$ denote @rclength. The $2^\text{nd}$ angle condition suggests projecting $C,D$ through $P$ to the circle at $C',D'$, so that $A@D'=B@C'$ and $ABD'C'$ is a CyclicISLscelesTrapezoid. Now, rewrite the angles in the $1^\text{st}$ condition:
\begin{align*}
\angle ADB&=\tfrac{1}{2}A@B=\tfrac{1}{2}(A@C'+B@D'+C'@D')\\
\angle CPD&=\tfrac{1}{2}(C@D+C@D')=\tfrac{1}{2}(C@M+D@M+C'@D').
\end{align*}Equating, we see that the $4$ @rclengths $A@C'$, $B@D'$, $C@M$, and $D@M$ are all equal. Besides the aforementioned $ABD'C'$ and the degenerate $CDMM$, these produce $2\cdot2=4$ or $\tbinom{4}{2}-2=4$ more CyclicISLscelesTrapezoids:
  1. $AC'CM$, which gives $AM\parallel C'P$;
  2. $AC'MD$, which gives $AD\parallel C'M$;
  3. $BD'DM$, which gives $BM\parallel D'P$; and
  4. $BD'MC$, which gives $BC\parallel D'M$.
Together, these are enough to show that $PMC'D'$ is homothetic to $MXAB$, where $X:=AD\cap BC$. $\blacksquare$

Remark. Furthermore, ray $XMP$ is concurrent with rays $AC'$ and $BD'$ at a point $Y$, since $Y$ is the $5^\text{th}$ vertex of the nested similar simplices $YPMC'D'$ and $YMXAB$ (more precisely, the projection onto the plane at hand). Hey, it isn't my fault the diagram looks hyperdimensional :ddr:
Z K Y
The post below has been deleted. Click to close.
This post has been deleted. Click here to see post.
Ilikeminecraft
637 posts
#38
Y by
I stared at this for 2 hours in geogebra before finally getting it.
Let $X = AD\cap BC.$
Claim: $DB, DC$ are isogonal with respect to $\angle FDM.$
Proof: Note that $\angle BDM = \angle MCX.$ Also note that $\angle PDB = \angle ACP$ from the first condition. Also note that $\angle MDC = \angle DCM$ from arc midpoint definition.
Note that $\angle ACX = \angle ACM +  \angle MCX = (\angle ACD + \angle MCD) + (\angle BDM) = \angle ACD + \angle BDC + 2\angle CDM.$ We also know that $\angle CPD = \angle ACB,$ so $\angle PDC + \angle PCD = \angle ACX.$ Thus, $\angle DCM = \angle PCA$ which finishes.

With this, note that $\angle PDB = \angle CDM = \angle MCD = \angle MBD,$ so $PD\parallel BM.$ Let $R=BM\cap AX, S = BX\cap AM.$ Observe that $SR\parallel CD$ since $BSRA$ is cyclic. Thus, $PCD, MSR$ are homothetic, which finishes.
Z K Y
The post below has been deleted. Click to close.
This post has been deleted. Click here to see post.
wu2481632
4239 posts
#39
Y by
Let $X$ be the intersection of $AM$ and $BP$, and let $Y$ be the intersection of $DM$ and $CP$.

Then $\angle ABX = \angle DCX$, and by the midpoint of arc condition, $\angle BAX = \angle CDY$, so triangles $ABX$ and $CDY$ are similar. The condition that $\angle ADB = \angle CPD$ and $\angle ADP = \angle PCB$ then yields that $PXMY$ is a parallelogram.

Suppose that $BP$ and $CP$ intersect $(ABCD)$ again at $V$ and $U$, respectively. Then let $AU$ and $DV$ meet at $Q$. Note that $AUVD$ is a trapezoid, so $UV \parallel AD$, and thus triangles $UVP$ and $ADM$ are homothetic with center $Q$. Thus $Q, P, M$ are collinear.

But by Pascal applied to hexagon $UABCDV$ it follows that $Q, P,$ and the intersection of $AB$ and $CD$ are collinear. Thus $AD, PM,$ and $BC$ are concurrent.
Z K Y
The post below has been deleted. Click to close.
This post has been deleted. Click here to see post.
ItsBesi
146 posts
#40
Y by
Nice problem.
Let $DA \cap CP=\{K\}$ , $CB \cap DP=\{L\}$ and $\odot(ABCMD)=\omega$

Claim:Points $K$,$L$,$C$ and $D$ are concyclic
Proof:
$\angle KDL \equiv \angle ADP=\angle PCB=\angle KCL \implies \angle KDL=\angle KCL \implies$ Points $K$,$L$,$C$ and $D$ are concyclic $\square$

Claim:$AB \parallel KL$
Proof:
Since both $\odot(ADCB)$ and $\odot(KLCD)$ are concyclic and both points $\overline{D-A-K}$ and $\overline{C-B-L}$ are collinear
we get by Reim's Theorem that $AB \parallel KL$ $\square$

Claim:$AM \parallel KP$ and $BM \parallel LP$
Proof:
Let $AM \cap DP=\{X\}$ and $BM \cap CP=\{Y\}$

$\angle PXM=\angle AXD \stackrel{\triangle AXD}{=} 180-\angle ADX-\angle DAX \equiv 180-\angle KDL-\angle DAM=180-\angle KDL-\angle MAC \stackrel{\omega}{=} 180-\angle KDL-\angle MBC$
$\stackrel{\odot(KLCD)}{=} 180-\angle KCL-\angle MCB \equiv 180-\angle YCB-\angle YBC \stackrel{\triangle YBC}{=}\angle BYC=\angle PYM \implies \angle PXM=\angle PYM ...(1)$

Also $\angle XPY \equiv \angle DPC=\angle ADB \stackrel{\omega}{=} \angle AMB \equiv \angle XMY \implies \angle XPY=\angle XMY ...(2)$

By combining $(1)$ and $(2)$ we get that the quadrilateral $PXMY$-is a parallelogram $XP \parallel PY$ and $YP \parallel PX \implies$
$AM \parallel KP$ and $BM \parallel LP$ $\square$

Claim: Lines $AD$,$PM$ and $BC$ are concurrent.
Proof:

Since $AB \parallel KL$, $AM \parallel KP$ and $BM \parallel BM$ we get that triangles $\triangle AMB$ and $\triangle KPL$ are homothetic $\implies$
Lines $KA$,$PM$ and $LB$ meet at one point $\iff$ Lines $AD$,$PM$ and $BC$ are concurrent $\blacksquare$
Attachments:
Z K Y
N Quick Reply
G
H
=
a